3f63004920d8d9e8a41ce35b75abe2cd8bf2bfd9
[course.git] / latex / problems / Serway_and_Jewett_8 / problem23.09.tex
1 \begin{problem*}{23.9}
2 Thre point charges are arranged as shown in Figure~P23.9.
3 Find \Part{a} the magnitude and \Part{b} the direction of the electric
4 force on the particle at the origin.
5 % y5.00nC      6.00nC
6 % o-----------o----x
7 % |0.100m  0.300m
8 % o-3.00nC
9 \end{problem*}
10
11 \begin{solution}
12 The electric field at the origin due to the other two charges is
13 \begin{equation}
14   \vect{E}_0 =   k\frac{6.00\U{nC}}{0.300\U{m}^2}(-\ihat)
15                + k\frac{-3.00\U{nC}}{0.100\U{m}^2}\jhat
16              = (-599.3\ihat - 2697\jhat)\U{N/C}
17 \end{equation}
18 The electric force on the particle at the origin can be found from the
19 electric field
20 \begin{equation}
21   \vect{F}_0 = q_0\vect{E}_0
22              = 5.00\U{nC}\cdot(-599.3\ihat - 2697\jhat)\U{N/C}
23              = (-2.997\E{-6}\ihat - 1.348\E{-5}\jhat)\U{N}
24 \end{equation}
25
26 \Part{a}
27 Using the Pythagorean theorem, the magnitude is
28 \begin{equation}
29   |\vect{F}_0| = \sqrt{F_{0,x}^2 + F_{0,y}^2}
30     = \ans{1.38\E{-5}\U{N}}
31 \end{equation}
32
33 \Part{b}
34 Using basic trig, the angle is
35 \begin{equation}
36   \theta = 180\dg + \arctan\p({\frac{F_{0,y}}{F_{0,x}}})
37     = 180\dg + 77.47\dg = \ans{257\dg}
38 \end{equation}
39 counter clockwise from the $x$ axis, where the $180\dg$ adjusts for
40 the back-side ($x<0$) arctangent.
41 \end{solution}